LSAT and Law School Admissions Forum

Get expert LSAT preparation and law school admissions advice from PowerScore Test Preparation.

User avatar
 Dave Killoran
PowerScore Staff
  • PowerScore Staff
  • Posts: 5852
  • Joined: Mar 25, 2011
|
#41329
Complete Question Explanation
(The complete setup for this game can be found here: lsat/viewtopic.php?t=8467)

The correct answer choice is (A)

After completing the first two questions with relative ease, you should arrive at this question feeling confident. Since this is a Local question, you should as always make a mini-diagram next to the question. The “if” statement in the question, in combination with the second rule, produces the following setup:

pt2_o91_g2_q8.png

Several of the answer choices can be eliminated by using the rule that states that each floor has either one or two apartments. Since both J and P live on the fourth floor, no other residents can live on the fourth floor, and answer choices (B) and (D) can be eliminated. Since K lives on the fifth floor, the M and N block cannot live on the fifth floor, and answer choice (C) can be eliminated. Answer choice (E) can be eliminated since P must live on the fourth floor. Accordingly, answer choice (A) is correct.
You do not have the required permissions to view the files attached to this post.
 aessy77
  • Posts: 1
  • Joined: Jul 25, 2017
|
#37644
Hi,

I set two diagrams for this question. (1) J4 -> K5 -> P4 and (2) P4X -> K5X -> J4X.

According to the first diagram, the answer should be (A), but according to the second diagram:

5 M N
4 K Q
3 P J
2 L
1 O

So the answer (A), (B), (C), and (E) all make sense.

Can you please help me solving this problem?

Thank you.
User avatar
 Jonathan Evans
PowerScore Staff
  • PowerScore Staff
  • Posts: 726
  • Joined: Jun 09, 2016
|
#37680
Hi, aessy,

Let's look at the two ways that this could work. The deciding factor will be the location of Q.

Start with the core setup for this local question:
  • 5 K
    4 J _
    3 _
    2 _
    1 _
We know that J must be on a floor with exactly one other and P is immediately beneath K. Therefore we know:
  • 5 K
    4 J P
    3 _
    2 _
    1 _
Now the next most restricted item is Q because Q cannot be on the 1st or 2nd floors. Floor 4 is finished with J and P, so Q must be on either 5 or 3. Let's look at both:
  • Scenario α: Q on 5
    5 K Q
    4 J P
    3 _
    2 _
    1 _

    Scenario ß: Q on 3
    5 K
    4 J P
    3 Q
    2 _
    1 _
Since we have a 2 – 2 – 2 – 1 – 1 distribution, let's consider the implications of this distribution on both scenarios:
  • Scenario α: Q on 5
    5 K Q
    4 J P
    3 _ → (one or two slots)
    2 _ → (exactly one slot, per the rule)
    1 _ → (one or two slots)

    Either 3 or 1 must have M ↔ N in it. This leaves us with 0 and L. L is by itself. It could be in 3, 2, or 1 by itself. O is in the remaining floor by itself.

    Thus, we have to Hurdle the Uncertainty™ for floors 1, 2, and 3. The items on these floors will be M & N, L, or O with the restriction that M & N cannot be on the 2nd floor.

    Scenario ß: Q on 3
    5 K O
    4 J P
    3 Q
    2 L
    1 M N

    In this scenario, M & N still must be together. Since the 2nd floor has exactly one apartment, M & N must be on the remaining empty floor, the 1st floor. L needs to be by itself, so the only option for L is the 2nd floor. O :dblline: Q so O must be with K on the 5th floor.
Now we can note that only answer choice (A) could be true.

Good question. I hope this helps!

Get the most out of your LSAT Prep Plus subscription.

Analyze and track your performance with our Testing and Analytics Package.